Sie sind auf Seite 1von 47

Chapter 2

2.1

Consider the signals displayed in Figure P2.1.Show that each of these signals can be
expressed as the sum of rectangular (t ) and/or triangular (t ) pulses.

Figure P2.1

Solution:

a.

t
t
x1 (t ) = +
2
4

t 3
t 3
b. x2 (t ) = 2

6
2
c.

11
7
3
1
5
9
x3 (t ) = ... + t + + t + + t + + t + t + t + ...
2

2
2
2
2
2
=

t ( 2n + 0.5)

n =

1
2012 by McGraw-Hill Education. This is proprietary material solely for authorized instructor use. Not authorized for sale or distribution in any
manner. This document may not be copied, scanned, duplicated, forwarded, distributed, or posted on a website, in whole or part.

t+4
t
t4
d. x4 (t ) = ... +
+ +
+ ...
2
2
2

t 4n
=

2
n =
2.2 For the signal x2 (t ) in Figure P2.1 (b) plot the following signals
x2 (t 3)

a.

b. x2 (t )
c.

x2 (2t )

d. x2 (3 2t )
Solution:

x2 (t 3)

x2 (t )

0
1 2 3 4 5 6 7 8 9

6 5 4 3 2 1

t
1 2 3

x2 (3 2t )

x2 (2t )

1
0

1 2 3 4 5 6

0
21

1 2 3 4

2.3 Plot the following signals


a.

x1 (t ) = 2 (t / 2) cos(6 t )

1 1

b. x2 (t ) = 2 + sgn ( t )
2 2

2
2012 by McGraw-Hill Education. This is proprietary material solely for authorized instructor use. Not authorized for sale or distribution in any
manner. This document may not be copied, scanned, duplicated, forwarded, distributed, or posted on a website, in whole or part.

x3 (t ) = x2 (t + 2)

c.

d. x4 (t ) = sinc ( 2t ) ( t / 2 )
Solution:
x2 (t )

2
1.5

x1(t)

1
0.5

t
1 2 3 4 5 6

-0.5
-1
1
-1.5
0.8

-2
-1

-0.8

-0.6

-0.4

-0.2

0.2

0.4

0.6

0.8

1
0.6

xx4(t)
(t)

tt

0.4

x3 (t )

0.2

2
0

-0.2

-0.4
-1

1 2 3 4

-0.8

-0.6

-0.4

-0.2

0.2

0.4

0.6

0.8

tt

2.4 Determine whether the following signals are periodic. For periodic signals,
determine the fundamental period.
a.

x1 (t ) = sin( t ) + 5cos(4 t / 5)

Solution:

sin( t ) is periodic with period T1 =

= 2 . cos(4 t / 5) is periodic with period

T
2
5
= . x1 (t ) is periodic if the ratio 1 can be written as ratio of
4 / 5 2
T2
integers. In the present case,
T2 =

T1 2 2 4
=
=
T2
5
5

3
2012 by McGraw-Hill Education. This is proprietary material solely for authorized instructor use. Not authorized for sale or distribution in any
manner. This document may not be copied, scanned, duplicated, forwarded, distributed, or posted on a website, in whole or part.

Therefore, x1 (t ) is periodic with fundamental period To such that


To = 5T1 = 4T2 = 10
b. x2 (t ) = e j 3t + e j 9t + cos(12t )
Solution:

2
. Similarly, e j 9t is periodic with period
3
2
2
T2 =
. cos(12t ) is periodic with period T3 =
= . x2 (t ) is periodic with
9
12 6
2
fundamental period To = LCM (T1 , T2 , T3 ) =
.
3

e j 3t is periodic with period T1 =

c.

x3 (t ) = sin(2 t ) + cos(10t )

Solution:

2
= 1 . cos(10t ) is periodic with period
2
T
2
T2 =
= . x3 (t ) is periodic if the ratio 1 can be written as ratio of integers.
10 5
T2
In the present case,
sin(2 t ) is periodic with period T1 =

T1 1 5 5
=
=
T2

Since is an irrational number, the ratio is not rational. Therefore, x3 (t ) is not


periodic.

d. x4 (t ) = cos 2 t + sin(5 t )
4

Solution:

cos 2 t is periodic with period T1 =


= 1 . sin(5 t ) is periodic with
4
2

T
2 2
period T2 =
= . x4 (t ) is periodic if the ratio 1 can be written as ratio of
T2
5 5
integers. In the present case,

4
2012 by McGraw-Hill Education. This is proprietary material solely for authorized instructor use. Not authorized for sale or distribution in any
manner. This document may not be copied, scanned, duplicated, forwarded, distributed, or posted on a website, in whole or part.

T1 1 5 5
=
=
T2
2
2
Therefore, x4 (t ) is periodic with fundamental period To such that
To = 2T1 = 5T2 = 2
2.5 Classify the following signals as odd or even or neither.
a.

x(t ) = 4t

Solution:

x(t ) = 4t = (4t ) = x(t ) . So x(t ) is odd.

b. x(t ) = e

Solution:
e

c.

=e

. So x(t ) is even.

x(t ) = 5cos(3t )

Solution:

Since cos(t ) is even, 5cos(3t ) is also even.

d. x(t ) = sin(3t )
2
Solution:

x(t ) = sin(3t ) = cos(3t ) which is even.


2
e. x(t ) = u (t )
Solution:

u (t ) is neither even nor odd; For example, u (1) = 1 but u (1) = 0 u (1) .

5
2012 by McGraw-Hill Education. This is proprietary material solely for authorized instructor use. Not authorized for sale or distribution in any
manner. This document may not be copied, scanned, duplicated, forwarded, distributed, or posted on a website, in whole or part.

f.

x(t ) = sin(2t ) + cos(2t )

Solution:

x(t ) is neither even nor odd; For example,


x( / 8) = 2 but x( / 8) = 0 x( / 8) .
2.6 Determine whether the following signals are energy or power, or neither and
calculate the corresponding energy or power in the signal:
a. x1 (t ) = u (t )
Solution:

The normalized average power of a signal x1 (t ) is defined as


T /2

1
1
2
u (t ) dt = lim

T T
T T
T /2

Px1 = lim

T /2

1T 1
=
T
2 2

1 dt = lim T
0

Therefore, x1 (t ) is a power signal.


b. x2 (t ) = 4 cos(2 t ) + 3cos ( 4 t )
Solution:

4 cos(2 t ) is a power signal. 3cos ( 4 t ) is also a power signal. Since x2 (t ) is


sum of two power signals, it is a power signal.
T /2

T /2

2
2
1
1
4 cos(2 t ) + 3cos ( 4 t ) dt = lim
4 cos(2 t ) + 3cos ( 4 t ) dt
Px2 = lim

T T
T T
T /2
T /2
T /2

1
16 cos 2 (2 t ) + 9 cos 2 ( 4 t ) + 24 cos(2 t ) cos ( 4 t ) dt
T T
T /2

= lim
Now
T /2

T /2

1
8
8T
=8
16 cos 2 (2 t )dt = lim
1 + cos(4 t ) ]dt = lim
[

T T
T T
T T
T /2
T /2
lim

T /2

1
lim
9 cos 2 (4 t )dt = 4.5
T T
T /2

6
2012 by McGraw-Hill Education. This is proprietary material solely for authorized instructor use. Not authorized for sale or distribution in any
manner. This document may not be copied, scanned, duplicated, forwarded, distributed, or posted on a website, in whole or part.

T /2

T /2

24
24
cos(2 t ) cos ( 4 t )dt = lim

cos(6 t ) + cos ( 2 t )dt = 0


T T
T T
T /2
T /2
lim

Therefore,
Px2 = 8 + 4.5 = 12.5
c.

x3 (t ) =

1
t

Solution:

1 T /2
2
2
t
dt
lim
=

= Tlim

=0
T
T
T

t
T
/
2

T
/2

T /2
T /2

T /2
T /2
T /2

1
1
1 1
1 2
2
2
Px3 = lim
t
dt
t
dt
1/
lim
lim
=
=

= lim

=0

T T
T T
T T
T
T T / 2
T /2
T /2
t T /2
T /2

Ex3 = lim

T /2

1/ t dt = lim
2

x3 (t ) is neither an energy nor a power signal.


d. x4 (t ) = e t u (t )
Solution:
T /2

Ex4 = lim

T /2

e u (t ) dt = lim

T /2

2 t

e 2 t
dt = lim
T
2

T /2

1
1
=
lim ( e T + 1) =
2 T
2

Thus x4 (t ) is an energy signal.


e.

x5 (t ) = (t / 3) + (t )

Solution:
x5 (t )

3 / 2

3/ 2

7
2012 by McGraw-Hill Education. This is proprietary material solely for authorized instructor use. Not authorized for sale or distribution in any
manner. This document may not be copied, scanned, duplicated, forwarded, distributed, or posted on a website, in whole or part.

T /2

Ex5 = lim

(t / 3) + (t ) dt =
2

T /2

1/2

1/ 2

3/ 2

1/2

1dt +

1/ 2

4dt +

1dt

3/2

= 1+ 4 +1 = 6
Thus x5 (t ) is an energy signal.
f.

x6 (t ) = 5e( 2t + j10 t )u (t )

Solution:
T /2

Ex6 = lim

5e( 2t + j10 t )u (t ) dt = lim

T /2

e
= 25 lim
T
4

4 t T /2

T /2

T /2

5e 2t e j10 t ) dt = lim 25 e4t dt


2

25
25
=
lim ( e 2T + 1) =

T
4
4

Thus x6 (t ) is an energy signal.


g. x7 (t ) =

( t 4n ) / 2

n =

Solution:

x7 (t ) is a periodic signal with period To = 4 .


1
Px7 =
To

To / 2

2
( t / 2 ) dt =

To
To / 2
2

1
2
0 (1 t ) dt = 2 0 1 2t + t dt
2

t3
1
1
1 1
= t t 2 + = 1 1 + =
2
3 0 2
3 6
2.7 Evaluate the following expressions by using the properties of the delta function:
a.

x1 (t ) = (4t ) sin(2t )

Solution:

1
4

(4t ) = (t )
1
1
x1 (t ) = (t ) sin(2t ) = (t ) sin(0) = 0
4
4

8
2012 by McGraw-Hill Education. This is proprietary material solely for authorized instructor use. Not authorized for sale or distribution in any
manner. This document may not be copied, scanned, duplicated, forwarded, distributed, or posted on a website, in whole or part.

b. x2 (t ) = (t ) cos 30 t +
4

Solution:


x2 (t ) = (t ) cos 30 t + = (t ) cos 0 + = cos (t )
4
4

4
c. x3 (t ) = (t )sinc(t + 1)
Solution:

x3 (t ) = (t )sinc(t + 1) = (t )sinc(0 + 1) = (t )sinc(1) = (t ) 0 = 0


d. x4 (t ) = (t 2)e t sin(2.5 t )
Solution:
x4 (t ) = (t 2)e t sin(2.5 t )
= (t 2)e 2sin(2.5 2) = (t 2)e 2sin( ) = 0

e.

(2t )sinc(t )dt

x5 (t ) =

Solution:

1
x5 (t ) = (2t )sinc(t )dt = (t )sinc(t ) dt
2

1
1
1
(t )sinc(0)dt = (t )dt =

2
2
2

f. x6 (t ) =

(t 3) cos(t )dt

Solution:

x6 (t ) =

(t 3) cos(t )dt = cos(3) (t 3)dt = cos(3)

9
2012 by McGraw-Hill Education. This is proprietary material solely for authorized instructor use. Not authorized for sale or distribution in any
manner. This document may not be copied, scanned, duplicated, forwarded, distributed, or posted on a website, in whole or part.

(2 t ) 1 t

g. x7 (t ) =

dt

Solution:

x7 (t ) =

(2 t )

1
1
dt = (t 2)
dt
3
1 t
1 t3

1
1
1
dt = (t 2)dt =
= (t 2)
3
1 2
7
7

h. x8 (t ) =

(3t 4)e

3t

dt

Solution:

x8 (t ) =

3t
(3t 4)e dt =

1
4
t e 3t dt

3 3

4
1
e 4
4 3
= t e 3 dt =
3 3
3

i.

e 4

t 3 dt = 3

x9 (t ) = '(t ) (t )

Solution:

x9 (t ) =

(t ) '( )d = (1) d (t )

=0

= (t ) = ( t + 0.5 ) ( t 0.5 )
'

2.8 For each of the following continuous-time systems, determine whether or not the system is (1)
linear, (2) time-invariant, (3) memoryless, and (4) casual.
a.

y (t ) = x(t 1)

Solution:

The system is linear, time-invariant, causal, and has memory. The system has
memory because current value of the output depends on the previous value of
the input. The system is causal because current value of the output does not
depend on future inputs. To prove linearity, let x(t ) = x1 (t ) + x2 (t ) . The
response of the system to x(t ) is

10
2012 by McGraw-Hill Education. This is proprietary material solely for authorized instructor use. Not authorized for sale or distribution in any
manner. This document may not be copied, scanned, duplicated, forwarded, distributed, or posted on a website, in whole or part.

y (t ) = x(t 1) = x1 (t 1) + x2 (t 1)
= y1 (t ) + y2 (t )
T
T
where x1 (t )
y1 (t ) and x2 (t )
y2 (t ) .

To show that the system is time-invariant, let x1 (t ) = x(t to ) be the system


input. The corresponding output is
y1 (t ) = x1 (t 1) = x(t 1 to ) = y (t to )
b. y (t ) = 3x(t ) 2
Solution:

The system is nonlinear, time-invariant, causal, and memoryless. The system is


memoryless because current value of the output depends only on the current
value of the input. The system is causal because current value of the output does
not depend on future inputs.
To prove nonlinearity, let x(t ) = x1 (t ) + x2 (t ) . The response of the system
to x(t ) is
y (t ) = 3x(t ) 2 = 3 [ x1 (t ) + x2 (t ) ] 2
y1 (t ) + y2 (t ) = [3 x1 (t ) 2] + [3 x2 (t ) 2]
To show that the system is time-invariant, let x1 (t ) = x(t to ) be the system
input. The corresponding output is
y1 (t ) = 3x1 (t ) 2 = 3x(t to ) 2 = y (t to )
c.

y (t ) = x(t )

Solution:

The system is nonlinear, time-invariant, causal, and memoryless. The system is


memoryless because current value of the output depends only on the current
value of the input. The system is causal because current value of the output does
not depend on future inputs.
To prove nonlinearity, let x(t ) = x1 (t ) + x2 (t ) . The response of the system to
x(t ) is

11
2012 by McGraw-Hill Education. This is proprietary material solely for authorized instructor use. Not authorized for sale or distribution in any
manner. This document may not be copied, scanned, duplicated, forwarded, distributed, or posted on a website, in whole or part.

y (t ) = x1 (t ) + x2 (t ) x1 (t ) + x2 (t )

Because

x1 (t ) + x2 (t ) = x1 (t ) + x2 (t ) y1 (t ) + y2 (t ) for all and ,


the system is nonlinear.
To show that the system is time-invariant, let x1 (t ) = x(t to ) be the system
input. The corresponding output is
y1 (t ) = x1 (t ) = x(t to ) = y (t to )

d. y (t ) = [ cos(2t )] x(t )
Solution:

The system is linear, time-variant, causal, and memoryless. The system is


memoryless because current value of the output depends only on the current
value of the input. The system is causal because current value of the output does
not depend on future inputs.
To prove nonlinearity, let x(t ) = x1 (t ) + x2 (t ) . The response of the system
to x(t ) is
y (t ) = [ cos(2t ) ] x(t ) = [ cos(2t ) ][ x1 (t ) + x2 (t ) ]
= [ cos(2t ) ] x1 (t ) + [ cos(2t ) ] x2 (t ) = y1 (t ) + y2 (t )
To prove time-variance, let x1 (t ) = x(t to ) be the system input. The
corresponding output is

y1 (t ) = [ cos(2t ) ] x1 (t ) = [ cos(2t ) ] x ( t to ) y (t to ) = cos 2 ( t to ) x ( t to )


e.

y (t ) = e x (t )

Solution:

The system is nonlinear, time-invariant, causal, and memoryless. The system is


memoryless because current value of the output depends only on the current
value of the input. The system is causal because current value of the output does
not depend on future inputs.

12
2012 by McGraw-Hill Education. This is proprietary material solely for authorized instructor use. Not authorized for sale or distribution in any
manner. This document may not be copied, scanned, duplicated, forwarded, distributed, or posted on a website, in whole or part.

To prove nonlinearity, let x(t ) = x1 (t ) + x2 (t ) . The response of the system


to x(t ) is
y (t ) = e x1 ( t ) + x2 ( t ) = e x1 ( t ) e x2 (t ) y1 (t ) + y2 (t ) = e x1 ( t ) + e x2 ( t )

for all and


To show that the system is time-invariant, let x1 (t ) = x(t to ) be the system
input. The corresponding output is
y1 (t ) = e x1 (t ) = e x (t to ) = y (t to )
f. y (t ) = tx(t )
Solution:

The system is linear, time-variant, causal, and memoryless. The system is


memoryless because current value of the output depends only on the current
value of the input. The system is causal because current value of the output does
not depend on future inputs. To prove nonlinearity, let x(t ) = x1 (t ) + x2 (t ) .
The response of the system to x(t ) is
y (t ) = t [ x1 (t ) + x2 (t ) ] = tx1 (t ) + tx2 (t ) = y1 (t ) + y2 (t )

To prove time-variance, let x1 (t ) = x(t to ) be the system input. The


corresponding output is
y1 (t ) = tx1 (t ) = tx ( t to ) y (t to ) = ( t to ) x ( t to )
t

g. y (t ) =

x(2 )d

Solution:

The system is linear, time-invariant, causal, and has memory. The system has
memory because current value of the output depends only on the past values of
the input. The system is causal because current value of the output does not
depend on future inputs. To prove linearity, let x(t ) = x1 (t ) + x2 (t ) . The
response of the system to x(t ) is

13
2012 by McGraw-Hill Education. This is proprietary material solely for authorized instructor use. Not authorized for sale or distribution in any
manner. This document may not be copied, scanned, duplicated, forwarded, distributed, or posted on a website, in whole or part.

[ x1 (2 ) + x2 (2 )] d = x1 (2 )d + x2 (2 )d = y1 (t ) + y2 (t )

y (t ) =

To check for time-variance, let x1 (t ) = x(t to ) be the system input. The


corresponding output is
t

y1 (t ) =

t to

x (2 )d = x [ 2( t )] d = x ( 2 ) d = y ( t t )
o

2.9 Calculate the output y (t ) of the LTI system for the following cases:
a.

x(t ) = e 2t u (t ) and h(t ) = u (t 2) u (t 4)

Solution:

t 3
h(t ) = u (t 2) u (t 4) =

2
y (t ) =

h( ) x(t )d = e

2( t )

3
u (t )
d
2

For t < 2 , there is no overlap and y (t ) = 0.


t

For 2 t < 4 , y (t ) = e

2( t )

d =

t 2

e 2
d =
2

t 2

1 e2(t 2)
=
2

For t 4 ,
4

y (t ) = e

2( t )

d = e

2 t

1 e 2(t 2)
,

2(t 2) ( e 4 1)
y (t ) = e
,
2

0,

d =e

2 t

e2
2

=e
2

2 t

e4 1)
e8 e 4
2( t 2) (
=e
2
2

2t 4
t>4
otherwise

b. x(t ) = e t u (t ) and h(t ) = e 2t u (t )

14
2012 by McGraw-Hill Education. This is proprietary material solely for authorized instructor use. Not authorized for sale or distribution in any
manner. This document may not be copied, scanned, duplicated, forwarded, distributed, or posted on a website, in whole or part.

Solution:

y (t ) =

u ( )e

2( t )

u (t )d = e e2(t ) d
0

For t < 0 , there is no overlap and y (t ) = 0.


y (t ) = e

2 t

e d =e

2 t

e = e2t ( et 1) = e t e 2t , t 0
t

c.

x(t ) = u (t ) and h(t ) = (t ) 3e2t u (t )

Solution:

y (t ) =

h( ) x(t )d = ( ) 3e 2 u ( ) u ( t )d

Now

( )u( t )d = u (t )

For t < 0 , 3e

For t 0 ,

3e

3e 2
d =
2

=
0

3
2

u ( )u ( t )d = 3e

5
3
2 + 1 = 2 ,
y (t ) =
3 e 2t ,
2

3e 2
d =
2

3
= e 2t
2

t<0
t0

d. x(t ) = (t 2) + 3e3t u (t ) and h(t ) = u (t ) u (t 1)


Solution:

15
2012 by McGraw-Hill Education. This is proprietary material solely for authorized instructor use. Not authorized for sale or distribution in any
manner. This document may not be copied, scanned, duplicated, forwarded, distributed, or posted on a website, in whole or part.

y (t ) =
=

h( ) x(t )d = (t 2) + 3e

3( t )

u (t + ) [u ( ) u ( 1)] d

3( t )
(t 2) [u ( ) u ( 1)] d + 3e u (t + ) [u( ) u ( 1)] d

Now

(t 2) [u( ) u( 1)] d = u(t 2) u(t 2 1) = ( t 2.5)

For t < 0 ,

3( t )
3( t )
3t
3
3e u(t + ) [u( ) u( 1)] d = 3e d = 3e e d
0

= 3e3t

3 1

e
3

= e 3t 1 3
e

For 0 < t 1 ,

3e

3( t )

u (t + ) [u ( ) u ( 1) ] d = 3e

3( t )

d = 3e

3t

e 3
1

= 3e
= e3t e 3t 3
3 t
e

3t

3t
1
e 1 e3 ,



1
y (t ) = e3t e 3t 3 ,
e

1,

0,

t0
0 < t 1
2t 3
otherwise

2.10 The impulse response function of a continuous-time LTI is displayed in Figure P2.2(b). Assuming
the input x(t ) to the system is waveform illustrated in Figure P2.2(a), determine the system output
waveform y (t ) and sketch it.
Solution:

For t < 1 , there is no overlap and y (t ) = 0.

16
2012 by McGraw-Hill Education. This is proprietary material solely for authorized instructor use. Not authorized for sale or distribution in any
manner. This document may not be copied, scanned, duplicated, forwarded, distributed, or posted on a website, in whole or part.

Figure P2.2
x( )

(a)

(b)

h(t )
1

1< t < 2

h(t )

(c)

5<t <6

3
t -1

As shown in Figure (b), y (t ) = d = t 1 for 1 t < 2


0

For 2 t < 3 , y (t ) = d = 1
0
1

d =1 t + 3 = 4 t

For 3 t < 4 , y (t ) =

t 3

Referring to Figure (c), y (t ) = d = t 5 for 5 t < 7


5

d = t t + 2 = 2

For 7 t < 8 , y (t ) =

t 2
7

For 8 t < 10 , y (t ) =

d = 7 t + 3 = 10 t

t 3

y(t )
2

t
10
17

2012 by McGraw-Hill Education. This is proprietary material solely for authorized instructor use. Not authorized for sale or distribution in any
manner. This document may not be copied, scanned, duplicated, forwarded, distributed, or posted on a website, in whole or part.

2.11 An LTI system has the impulse response h(t ) = e 0.5(t 2)u (t 2) .
a.

Is the system casual?

Solution:

Yes. The system is casual because h(t ) = 0 for t < 0 .


b. Is the system stable?
Solution:

The system is stable because

h(t ) dt =

e
0

0.5 x

e 0.5 x
dx =
0.5

=2
0

c. Repeat parts (a) and (b) for h(t ) = e 0.5(t + 2)u (t + 2) .


Solution:

The system is not causal but stable.


2.12 Write down the exponential Fourier series coefficients of the signal
x(t ) = 5sin(40 t ) + 7 cos ( 80 t / 2 ) cos (160 t + / 4 )

Solution:

Applying the Eulers formula, we get the following terms:


e j 40 t e j 40 t
e j 80 t e j /2 + e j 80 t e j /2 e j160 t e j / 4 + e j160 t e j /4
+
x(t ) = 5
7

2j
2
2

j 40 t
j 40 t
j 80 t
j 80 t
j /4 j160 t
j /4 j160 t
= j 2.5e
+ j 2.5e
j 3.5e
+ j 3.5e
0.5e e
0.5e
e

The Fourier coefficients are

C1 = j 2.5, C1 = j 2.5
C2 = j 3.5, C2 = j 3.5
C3 = 0.5e j /4 , C3 = 0.5e j /4
a. Is x(t) periodic? If so, what is its period?
Solution:

18
2012 by McGraw-Hill Education. This is proprietary material solely for authorized instructor use. Not authorized for sale or distribution in any
manner. This document may not be copied, scanned, duplicated, forwarded, distributed, or posted on a website, in whole or part.

Yes. o = 40 f o = 20, To =

1
1
. The period is
= 0.05sec .
20
20

2.13 A signal has the two-sided spectrum representation shown in Figure P2.3.
Figure P2.3

a. Write the equation for x(t).


Solution:

x(t ) = 14 cos 100 t + 10 + 8cos 300 t


3
2

b. Is the signal periodic? If so, what is its period?


Solution:

Yes. It is periodic with fundamental period To =

1
= 0.02 .
50

c. Does the signal have energy at DC?


Solution:

Yes as indicated by the presence of DC term C0 = 10 .


2.14 Write down the complex exponential Fourier series for each of the periodic signals shown in
Figure P2.4. Use odd or even symmetry whenever possible.
Solution:

a. To = 3, f o = 1/ 3

19
2012 by McGraw-Hill Education. This is proprietary material solely for authorized instructor use. Not authorized for sale or distribution in any
manner. This document may not be copied, scanned, duplicated, forwarded, distributed, or posted on a website, in whole or part.

2
3
1
1
C0 = 2dt 1dt = ( 4 1) = 1
3 0
2
3
2
3
2 nt
2 nt
j
j

1
Cn = 2e 3 dt e 3 dt
3 0
2

2
2 nt 3
j
j 3 j 23nt
2e
=
e 3
3 2 n
0
2

j
2e j 4 n /3 2 e j 2 n + e j 4 n /3
=
2 n
3e j 2 n /3 e + j 2 n /3 e j 2 n /3
j 3 j 4 n /3
=

=
1
e
(
) n

2 n
2j

3e j 2 n /3
2 n
=
sin
, n = 1, 2,.....
n
3

b. To = 2, f o = 1/ 2
1
1 t2
1
C0 = tdt =
2 1 2 2

Cn =

=
1

1
(1 1) = 0
4

1
1

1 j nt
1
j nt
=

te
dt
)

td ( e

2 1
j 2 n 1

j j nt 1
j nt

te
e
dt

1
2 n
1

1 j nt 1
j j n + j n
+e
+
e
e

1
2 n
j n

j e j n e + j n e j n
e j n
=
+
+ 2 2 2 2
2 n
j n
j n
n
je j n j ( 1)
=
=
, n = 1, 2,.....
n
n
n

c. To = 2, f o = 1/ 2
1

1
t2

2A
1
A
1
C0 =

t
dt
=
A
) t = A 1 =
(
2 0
2 0
2 2

20
2012 by McGraw-Hill Education. This is proprietary material solely for authorized instructor use. Not authorized for sale or distribution in any
manner. This document may not be copied, scanned, duplicated, forwarded, distributed, or posted on a website, in whole or part.

Since x3 (t ) is an even function of time,


1
1

An 2 A 1

1
cos(
)
cos(
)
t
nt
dt
A
nt
dt
A
=

(
)

0
0 t cos( nt )dt
2 2 0

1
1

A
A
1

sin(
)
sin(
)
sin( nt )dt
td
nt
dt
t
nt
= 0
=

+
(
)

0
n 0
n
0

1
A cos( nt )
A
A
0
=

= 2 2 [ cos( n) 1] = 2 2 [1 cos( n)]


n
n 0
n
n
n even
0,

= 2A
n odd
2 n 2 ,

Cn =

d. To = 3, f o = 1/ 3
1
1
3/ 2
1 1 1 2A
2A t2
2A
3/2
C0 =
+t1 = + =

tdt + 1dt =
3 0
3 2 2 3
1
3 2 0

Since x4 (t ) is an even function of time,


Cn =

1.5
3/2
2A 1

An 2 A
=
=
+
x
t
nt
dt
t
nt
dt
(
)
cos(2
/
3)
cos(2
/
3)
cos(2 nt / 3)dt

2 3 0
1
3 0

Now
1

1
1

3
3
1

td
sin(2
nt
/
3)
t
sin(2
nt
/
3)
sin(2 nt / 3)dt
=

(
)

0
2 n 0
0
2 n

3
3
1

sin(2
n
/
3)
cos(2 nt / 3) 0
=
+

2 n
2 n

3
3
sin(2 n / 3) +
=
[cos(2 n / 3) 1]

2 n
2 n

t cos(2 nt / 3)dt =
0

3/2

cos(2 nt / 3)dt = 2 n sin(2 nt / 3)


1

3/2
1

3
2 n

sin(2 n / 3)

Substituting yields

21
2012 by McGraw-Hill Education. This is proprietary material solely for authorized instructor use. Not authorized for sale or distribution in any
manner. This document may not be copied, scanned, duplicated, forwarded, distributed, or posted on a website, in whole or part.

Cn =

e.

2A 3

3 2 n

x5 (t ) =

2 n
3A
cos 3 1 = 2 2 n 2

2 n
cos 3 1

p ( t 8n )

n =

where
p ( t ) = ( t 2 ) / 2 + ( t 2 ) / 4 ( t 6 ) / 2 ( t 6 ) / 4
The FT of the pulse shape p ( t ) over [ 0, To ] is given by
To

P ( f ) = p (t )e j 2 ft dt
0

The FS coefficients of a periodic signal with basic pulse shape p ( t ) are given by
Cn =

1
To

To

p(t )e

j 2 nf o t

dt

Comparing yields
Cn =

1
P ( f ) f = nf
o
To

Now

( t 2 ) / 2
2sinc ( 2 f ) e j 4 f

( t 2 ) / 4
4sinc ( 4 f ) e j 4 f

( t 6 ) / 2
2sinc ( 2 f ) e j12 f

( t 6 ) / 4
4sinc ( 4 f ) e j12 f

Therefore,

P ( f ) = 2sinc ( 2 f ) e j 4 f 1 e j 8 f + 4sinc ( 4 f ) e j 4 f 1 e j 8 f
The FS coefficients of a periodic signal x5 (t ) are now obtained as

22
2012 by McGraw-Hill Education. This is proprietary material solely for authorized instructor use. Not authorized for sale or distribution in any
manner. This document may not be copied, scanned, duplicated, forwarded, distributed, or posted on a website, in whole or part.

Cn =
=

1
2sinc ( 2nf o ) e j 4 nfo 1 e j 8 nfo + 4sinc ( 4nf o ) e j 4 nfo 1 e j 8 nfo
To

1
sinc ( n / 4 ) e j n /2 1 e j n + 2sinc ( n / 2 ) e j n /2 1 e j n
4

2.15 For the rectangular pulse train in Figure 2.23, compute the Fourier coefficients of the new periodic
signal y(t) given by
a.

y (t ) = x(t 0.5To )

Solution:

The FS expansion of a periodic pulse train x(t ) =

( t nTo )
of

n =

rectangular pulses is given by

x(t ) =

C e

n =

x
n

j 2 nf o t

where the exponential FS coefficients are


Cnx =

To

sinc ( nf o )

Let the FS expansion of a periodic pulse train y (t ) = x(t 0.5To ) be expressed as


y (t ) =

n =

y
n

e j 2 nfot

where
Cny =
=

1
1
y (t )e j 2 nfot dt = x(t 0.5To )e j 2 nfot dt

To To
To To
1
j 2 nf o ( + 0.5To )
j 2 nf o ( 0.5To ) 1
x( )e
d = e
x( )e j 2 nfo d

To To
To To
Cnx

= e j nCnx

Time Shifting introduces a linear phase shift in the FS coefficients; their


magnitudes are not changed.

23
2012 by McGraw-Hill Education. This is proprietary material solely for authorized instructor use. Not authorized for sale or distribution in any
manner. This document may not be copied, scanned, duplicated, forwarded, distributed, or posted on a website, in whole or part.

b. y (t ) = x(t )e j 2 t / To
Solution:

Cny =
=

1
1
y (t )e j 2 nfot dt = x(t )e j 2 tfo e j 2 nfot dt

To To
To To
1
j 2 tf o ( n 1)
x(t )e
dt

To To

= Cnx1
c.

y (t ) = x( t )

Solution:
Cny =

1
1
y (t )e j 2 nfot dt =

To To
To

x ( t )e

j 2 nf o t

dt

To

j 2 n o
1

(
)
=
x

e
d
To To

= Cnx

Time Scaling does not change FS coefficients but the FS itself has changed as the
f
2f
3f
harmonic components are now at the frequencies o , o , o ,

2.16 Draw the one-sided power spectrum for the square wave in Figure P2.5 with duty cycle 50%.
Figure P2.5

Solution:

The FS expansion for the square wave is given by

24
2012 by McGraw-Hill Education. This is proprietary material solely for authorized instructor use. Not authorized for sale or distribution in any
manner. This document may not be copied, scanned, duplicated, forwarded, distributed, or posted on a website, in whole or part.

x(t ) =

Ce

n =

j 2 nf o t

where
1
Cn = x(t )e j 2 nfot dt
To To
To
To / 2 j 2 nf t

j 2 nf o t
o
A
e
dt

A
e
dt

To / 2
0

To / 2
To
A
=
e j 2 nfot
e j 2 nfot
0
To /2
To ( j 2 nf o )

1
=
To

jA j n
e
1 e j 2 n + e j n )
(
2 n

Therefore,
j2 A
,

Cn = n
0,

n odd
otherwise

Average power in the frequency component at f = nf o equals Cn . Figure


2

displays the one-sided power spectrum for the square wave.

One-sided Power Spectrum of Square wave


0.9
0.8

One-sided Power Spectrum

0.7
0.6
0.5
0.4
0.3
0.2
0.1
0

10
15
Frequency (xfo)

20

25

a. Calculate the normalized average power.


Solution:
25
2012 by McGraw-Hill Education. This is proprietary material solely for authorized instructor use. Not authorized for sale or distribution in any
manner. This document may not be copied, scanned, duplicated, forwarded, distributed, or posted on a website, in whole or part.

The normalized average power for a periodic signal x (t) is given by


1
Px =
To

To /2

x(t ) dt =
2

To /2

A2To
= A2
To

b. Determine the 98% power bandwidth of the pulse train.


Solution:

1
3
5
7
9
11
13
15
17
19
21
2.17

Total Power
including current
Fourier coefficient
0.8106
0.9006
0.9331
0.9496
0.9596
0.9663
0.9711
0.9747
0.9775
0.9798
0.9816

98% Power bandwidth = 21 f o

Determine the Fourier transforms of the signals shown in Figure P2.6.

Figure P2.6

26
2012 by McGraw-Hill Education. This is proprietary material solely for authorized instructor use. Not authorized for sale or distribution in any
manner. This document may not be copied, scanned, duplicated, forwarded, distributed, or posted on a website, in whole or part.

Solution (a)

The pulse x1 (t ) can be expressed as


x1 (t ) = A ( t + 0.5 ) ( t 0.5 )
Now

( t )
sinc ( f )

Applying the time-shifting property of the FT, we obtain

( t + 0.5 )
sinc ( f ) e j f

( t 0.5 )
sinc ( f ) e j f

Adding

X 1 ( f ) = A sinc ( f ) e j f sinc ( f ) e j f = Asinc ( f ) e j f e j f


= Aj 2sinc ( f ) sin ( f ) = j 2 fA sinc 2 ( f )
Solution (b)

The pulse x2 (t ) can be expressed as


x2 (t ) = ( 2t ) cos ( 2 t )

Now
1

(2t )
sinc( f / 2)
2
1

cos(2 t )
( f 1) + ( f + 1)
2
1
1
X 2 ( f ) = { (2t )} {cos(2 t )} = sinc( f / 2) ( f 1) + ( f + 1)
2
2
1
= sinc 0.5 ( f 1) + sinc 0.5 ( f + 1)
4

Solution (c)

The pulse x3 (t ) can be expressed as

27
2012 by McGraw-Hill Education. This is proprietary material solely for authorized instructor use. Not authorized for sale or distribution in any
manner. This document may not be copied, scanned, duplicated, forwarded, distributed, or posted on a website, in whole or part.

x3 (t ) = p ( t ) + p (t )

where
p ( t ) = e t u ( t ) u ( t 1)
From Table 2.2, we have

e t u (t )

1
1 + j 2 f

Using the time-shifting property of the FT, we obtain


e u ( t 1) = e e
t

1 ( t 1)

e j 2 f
e (1+ j 2 f )
=
u ( t 1) e
1 + j 2 f 1 + j 2 f

Combining

1
e (1+ j 2 f )
1
1 e(1+ j 2 f )
p(t ) P ( f ) =

1 + j 2 f 1 + j 2 f 1 + j 2 f

By time-reversal property,

p ( t )
P ( f ) =

1
1 e (1 j 2 f )

1 j 2 f

X3 ( f ) = P ( f ) + P ( f ) =
=
=

1
1
(1 j 2 f )
1 e(1+ j 2 f ) +

1 j 2 f 1 e

1 + j 2 f

{(1 j 2 f ) 1 e
1 + ( 2 f )
1

2
1 + ( 2 f )

(1+ j 2 f )

+ (1 + j 2 f ) 1 e(1 j 2 f )

1 e1 cos ( 2 f ) + 2 fe1 sin ( 2 f )

Solution (d)

The pulse x4 (t ) can be expressed as


x4 (t ) = ( t / 2 ) + ( t / 4 )

Now

28
2012 by McGraw-Hill Education. This is proprietary material solely for authorized instructor use. Not authorized for sale or distribution in any
manner. This document may not be copied, scanned, duplicated, forwarded, distributed, or posted on a website, in whole or part.


( t / 2 )
2sinc ( 2 f )

( t / 4 )
4sinc ( 4 f )

Adding
X 4 ( f ) = 2sinc ( 2 f ) + 4sinc ( 4 f )

2.18 Use properties of the Fourier transform to compute the Fourier transform of
following signals.
a. sinc 2 (Wt )
Solution:

(t / )
sinc 2

2
2
Using the duality property, we obtain

Wsinc 2 (Wt )
( f / 2W )

sinc 2 (Wt )

1
( f / 2W )
W

Thus the Fourier transform of a sinc2 pulse is a triangular function in frequency.


b. (t / T ) cos(2 f c t )
Solution:

( t / T )
Tsinc ( fT )

cos(2 f ct )
( f f c ) + ( f + f c )
2
1
X ( f ) = { ( t / T )} {cos(2 f c t )} = Tsinc ( fT ) ( f f c ) + ( f + f c )
2
T
sin c T ( f f c ) + sinc T ( f + f c )
=
2

c.

(e

cos10 t ) u (t )

Solution:

29
2012 by McGraw-Hill Education. This is proprietary material solely for authorized instructor use. Not authorized for sale or distribution in any
manner. This document may not be copied, scanned, duplicated, forwarded, distributed, or posted on a website, in whole or part.

From Table 2.2,

e t u (t )

1
1 + j 2 f

Now

} {

( e t u (t ) ) cos10 t = ( e t u (t ) ) {cos10 t}
=

1
1
( f 5 ) + ( f + 5 )
1 + j 2 f 2

1
1
1
+

2 1 + j 2 ( f 5 ) 1 + j 2 ( f + 5 )

1 1 + j 2 ( f + 5 ) + 1 + j 2 ( f 5 )

2
2
2
(1 + j 2 f ) ( j 2 5)

1 + j 2 f
=

2
2
(1 + j 2 f ) + 100

1 + j 2 f

=
2
2 2
(1 + 100 ) + j 4 f 4 f
d.

te t u (t )

Solution:

Applying the differentiation in frequency domain property in (2.85), we obtain

te t u (t )

j d
{e t u (t )}
2 df

That is,
1

j d (1 + j 2 f )
j
1
j 2
{te u (t )} =
=
2
2
df
2
(1 + j 2 f )
t

e. e t

(1 + j 2 f )

Solution:

30
2012 by McGraw-Hill Education. This is proprietary material solely for authorized instructor use. Not authorized for sale or distribution in any
manner. This document may not be copied, scanned, duplicated, forwarded, distributed, or posted on a website, in whole or part.

X(f ) =

t 2

j 2 ft

dt =

t 2 + j 2 ft

) dt

Multiplying the right hand side by e f e f yields


2

X(f ) =e

f 2

(
e

t 2 + j 2 ft f 2

) dt = e f

( t + jf )
dt
e
2

Substituting t + jf = , we obtain
X(f ) =e

f 2

e d
2

f.

4sinc (t ) cos (100 t )


2

Solution:

4sinc 2 ( t )
4
2
1

cos (100 t )
( f 50 ) + ( f + 50 )
2
f 1
X ( f ) = {4sinc 2 ( t )} {cos (100 t )} = 4 ( f 50 ) + ( f + 50 )
2 2

= 2 0.5 ( f 50 ) + 0.5 ( f 50 )
2.19 Find the following convolutions:
a. sinc(Wt ) sinc(2Wt )
Solution:

We use the convolution property of Fourier transform in (2.79).

x(t ) y (t )
X ( f )Y ( f )

Now

sinc(2Wt )

1
( f / 2W )
2W

31
2012 by McGraw-Hill Education. This is proprietary material solely for authorized instructor use. Not authorized for sale or distribution in any
manner. This document may not be copied, scanned, duplicated, forwarded, distributed, or posted on a website, in whole or part.

Therefore,
1
1
( f / W )
( f / 2W )
2W
W
1
=
( f / W )
2W 2

sinc(Wt ) sinc(2Wt )

b. sinc 2 (Wt ) sinc(2Wt )


Solution:

Again using the convolution property of Fourier transform


1
1
( f / 2W )
( f / 2W )
2W
W
1
=
( f / 2W )
2W 2

sinc 2 (Wt ) sinc(2Wt )

2.20 The FT of a signal x(t ) is described by


X(f ) =

1
5 + j 2 f

Determine the FT V ( f ) of the following signals:


a. v(t ) = x(5t 1)
Solution:

1
v(t ) = x(5t 1) = x 5 t
5
Let

1 f 1
1
X =
5 5 5 ( j 2 f / 5 ) + 5
1
=
j 2 f + 25

y (t ) = x(5t )
Y ( f ) =

j 2 f
j 2 f
1
1
5
=
v(t ) = y t V ( f ) = Y ( f )e
e 5
j 2 f + 25
5

32
2012 by McGraw-Hill Education. This is proprietary material solely for authorized instructor use. Not authorized for sale or distribution in any
manner. This document may not be copied, scanned, duplicated, forwarded, distributed, or posted on a website, in whole or part.

b. v(t ) = x(t ) cos(100 t )


Solution:

x(t )e j100 t + x(t )e j100 t


v(t ) = x(t ) cos(100 t ) =
2
Now
x(t )e j100 t + x(t )e j100 t
1
V ( f ) = X ( f 50 ) + X ( f + 50 )
2
2
1
1
=
+
j 2 ( f 50 ) + 5 j 2 ( f + 50 ) + 5
After simplification, we get
V( f ) =

j 2 f + 5
j 20 f + ( 2104 + 25 4 2 f 2 )

c. v(t ) = x(t )e j10t


Solution:
5
1

V ( f ) = X f =
v(t ) = x(t )e j10t
5

j 2 f + 5

dx(t )
d. v(t ) =
dt

Solution:

Using the differentiation property of FT


V ( f ) = j 2 fX ( f ) =

x(t )
j 2fX ( f ) , we obtain
dt

j 2 f
5 + j 2 f

e. v(t ) = x(t ) u (t )
Solution:

Using the convolution property of FT x(t ) y (t )


X ( f )Y ( f ) , we can write

33
2012 by McGraw-Hill Education. This is proprietary material solely for authorized instructor use. Not authorized for sale or distribution in any
manner. This document may not be copied, scanned, duplicated, forwarded, distributed, or posted on a website, in whole or part.

1
1
1
( f )+

j 2 f
5 + j 2 f 2
1
1
1
= (f )
+
2
5 + j 2 f j 2 f ( 5 + j 2 f )

V ( f ) = X ( f )U ( f ) =

1
1
(f )+
10
j 2 f ( 5 + j 2 f )

2.21 Consider the delay element y (t ) = x(t 3) .


a. What is the impulse response h(t)?
Solution:

Since y (t ) = h(t ) x(t ) = x(t 3) , the impulse response of the delay element is
given from (2.16) as
h(t ) = (t 3)
b. What is the magnitude and phase response function of the system?
Solution:

H ( f ) = { (t 3)} = e j 6 f
H( f ) =1
H ( f ) = 6 f
2.22 The periodic input x(t ) to an LTI system is displayed in Figure P2.7. The frequency
response function of the system is given by
H( f ) =

2
2 + j 2 f

Figure P2.7

34
2012 by McGraw-Hill Education. This is proprietary material solely for authorized instructor use. Not authorized for sale or distribution in any
manner. This document may not be copied, scanned, duplicated, forwarded, distributed, or posted on a website, in whole or part.

a. Write the complex exponential FS of input x(t ) .


Solution:

The input x(t ) is rectangular pulse train in Example 2.24 shifted by To / 4 and
duty cycle

= 0.5 . That is,

To

x(t ) = gTo ( t To / 4 ) =

( t nTo To / 4 )

0.5To

n =

The complex exponential FS of gTo (t ) from Example 2.24 with


To = 2 ( f o = 0.5 ) and

To

= 0.5 is given by

gTo (t ) = 0.5 sinc ( 0.5n )e j nt


n =

In Exercise 2.15(a), we showed that time shifting introduces a linear phase shift
in the FS coefficients; their magnitudes are not changed. The phase shift is equal
to e j 2 nfou for a time shift of u. The exponential FS coefficients x(t ) are
Cn = 0.5sinc ( 0.5n ) e

j 2 nf o ( 0.25To )

= e j n /2 0.5sinc ( 0.5n )

x(t ) = 0.5 e j n /2sinc ( 0.5n ) e j nt


n =

b. Plot the magnitude and phase response functions for H ( f ) .


Solution:

H( f ) =

2
1
=
2 + j 2 f 1 + j f

H ( nf o ) =

1
1
=
1 + j f 1 + j nf o

35
2012 by McGraw-Hill Education. This is proprietary material solely for authorized instructor use. Not authorized for sale or distribution in any
manner. This document may not be copied, scanned, duplicated, forwarded, distributed, or posted on a website, in whole or part.

0
-10

-5
Phase Response(degrees)

Magnitude Response(dB)

-20
-10

-15

-20

-30
-40
-50
-60
-70

-25
-80
-30

4
6
Frequency (f)

-90

4
6
Frequency (f)

c. Compute the complex exponential FS of the output y (t ) .


Solution:

Using (2.114), the output of an LTI system to the input

x(t ) =

0.5e

j n / 2

n =

sinc ( 0.5n ) e j nt
Cn

is given by

y (t ) =

C H (0.5n)e

j nt

n =

0.5e j n /2

sinc ( 0.5n ) e j nt
=
n = 1 + j 05 n

FS coefficient of y ( t )

2.23 The frequency response of an ideal LP filter is given by


5e j 0.0025 f ,
H( f ) =
0,

f < 1000 Hz
f > 1000 Hz

Determine the output signal in each of the following cases:


a.

x(t ) = 5sin ( 400 t ) + 2 cos (1200 t / 2 ) cos ( 2200 t + / 4 )

36
2012 by McGraw-Hill Education. This is proprietary material solely for authorized instructor use. Not authorized for sale or distribution in any
manner. This document may not be copied, scanned, duplicated, forwarded, distributed, or posted on a website, in whole or part.

Solution:

5,
H( f ) =
0,

0.0025 f ,
H( f ) =
0,
Since H (200) = 5 and

f < 1000 Hz
f > 1000 Hz
f < 1000 Hz
f > 1000 Hz

H (200) = / 2 , the output of the system for an input

5sin(400t) can now be expressed using (2.117) as 25sin(400t /2).


Next H (600) = 5 and

H (600) = 3 / 2 = / 2 , the output of the system for

an input 2 cos (1200 t / 2 ) can now be expressed using (2.117) as


10 cos (1200 t / 2 + / 2 ) = 10 cos (1200 t ) .

Now H (1100) = 0. So the LP filter doesnt pass cos ( 2200 t + / 4 ) . The


output of the LP filter is therefore given by
y (t ) = 25sin ( 400 t / 2 ) + 10 cos (1200 t )

b. x(t ) = 2sin(400 t ) +

sin ( 2200 t )
t

Solution:

Since H (200) = 5 and

H (200) = / 2 , the output of the system for an input

2sin(400t) is 10sin(400t /2).


To calculate the response to

sin ( 2200 t )
, we note that
t

sin ( 2200 t )
f

= 2200 sinc ( 2200t )

t
2200

In frequency domain, the output of LP filter to

sin ( 2200 t )
is
t

f j 0.0025 f f
f j 0.0025 f

5e
= 5
e
2200
2000
2000

37
2012 by McGraw-Hill Education. This is proprietary material solely for authorized instructor use. Not authorized for sale or distribution in any
manner. This document may not be copied, scanned, duplicated, forwarded, distributed, or posted on a website, in whole or part.

Now
f j 0.0025 f

2000sinc 2000 ( t 0.00125 )


e
2000
The output of the LP filter is therefore given by
y (t ) = 10sin ( 400 t / 2 ) + 10000sinc 2000 ( t 0.00125 )
c.

x(t ) = cos(400 t ) +

sin (1000 t )
t

Solution:

Since H (200) = 5 and

H (200) = / 2 , the output of the system for an input

cos(400 t ) is 5cos ( 400 t / 2 ) .


To calculate the response to

sin (1000 t )
, we note that
t

sin (1000 t )
f

= 1000 sinc (1000t )


t
1000

In frequency domain, the output of LP filter to

sin (1000 t )
is
t

f
f j 0.0025 f
5e j 0.0025 f
= 5
e
1000
1000
Now
f j 0.0025 f

1000sinc 1000 ( t 0.00125 )


e
1000
The output of the LP filter is therefore given by
y (t ) = 5cos ( 400 t / 2 ) + 5000sinc 1000 ( t 0.00125 )
d. x(t ) = 5cos(800 t ) + 2 (t )
Solution:

38
2012 by McGraw-Hill Education. This is proprietary material solely for authorized instructor use. Not authorized for sale or distribution in any
manner. This document may not be copied, scanned, duplicated, forwarded, distributed, or posted on a website, in whole or part.

Since H (400) = 5 and

H (400) = , the output of the system for an input

5cos ( 800 t ) is 25cos ( 800 t ) . The response of the system to (t ) is h(t ) .

The impulse response of the ideal LP filter is 10000sinc 2000 ( t 0.00125 ) .


Combining
y (t ) = 25cos ( 800 t ) + 2h(t )
= 25cos ( 800 t ) + 20 103 sinc 2000 ( t 0.00125 )
2.24 The frequency response of an ideal HP filter is given by
4,
H( f ) =
0,

f > 20 Hz,
f < 20 Hz

Determine the output signal y (t ) for the input


a.

x(t ) = 5 + 2 cos ( 50 t / 2 ) cos ( 75 t + / 4 )

Solution:
y (t ) = 8cos ( 50 t / 2 ) 4 cos ( 75 t + / 4 )

b. x(t ) = cos ( 20 t 3 / 4 ) + 3cos (100 t + / 4 )


Solution:
y (t ) = 12 cos (100 t + / 4 )

2.25 The frequency response of an ideal BP filter is given by


2e j 0.0005 f ,
900< f < 1000 Hz,
H( f ) =
otherwise
0,
Determine the output signal y (t ) for the input
a.

x(t ) = 2 cos (1850 t / 2 ) cos (1900 t + / 4 )

Solution:
H (925) = 2 and H (950) = 2 .

39
2012 by McGraw-Hill Education. This is proprietary material solely for authorized instructor use. Not authorized for sale or distribution in any
manner. This document may not be copied, scanned, duplicated, forwarded, distributed, or posted on a website, in whole or part.

H ( f ) = 0.0005 f . Therefore,
H (925) = 0.0005 f = 0.0005 925 = 0.462
H (950) = 0.0005 f = 0.0005 950 = 0.475
y (t ) = 4 cos (1850 t / 2 0.462 ) 2 cos (1900 t + / 4 0.475 )

b. x(t ) = sinc ( 60t ) cos(1900 t )


Solution:

X(f ) =
=

1 f 1
[ ( f 950) + ( f + 950) ]
60 60 2

1 f 950
f + 950

120 60
60

Y( f )= H( f )X ( f )
= 2e j 0.0005 f

1 f 950
f + 950

+

120 60
60

Now
1
f 950
j1900 t

sinc ( 60t ) e
60
60
e j 0.0005 f

1
f 950
j1900 ( t 0.00025 )

sinc 60 ( t 0.00025 ) e
60
60

Similarly
1
f + 950
j1900 t

sinc ( 60t ) e
60
60
e j 0.0005 f

1
f + 950
j1900 ( t 0.00025 )

sinc 60 ( t 0.00025 ) e
60
60

Therefore,

y (t ) = sinc 60 ( t 0.00025 ) e j1900 ( t 0.00025) + e j1900 ( t 0.00025)


= 2sinc 60 ( t 0.00025 ) cos 1900 ( t 0.00025 )

40
2012 by McGraw-Hill Education. This is proprietary material solely for authorized instructor use. Not authorized for sale or distribution in any
manner. This document may not be copied, scanned, duplicated, forwarded, distributed, or posted on a website, in whole or part.

c.

x(t ) = sinc 2 ( 30t ) cos(1900 t )

Solution:

X(f ) =
=

1 f 1
[ ( f 950) + ( f + 950) ]
30 60 2
1 f 950
f + 950

60 60
60

Y( f )= H( f )X ( f )
= 2e j 0.0005 f

1 f 950
f + 950

60 60
60

Now
1
f 950
2
j1900 t

sinc ( 30t ) e
30
60
e j 0.0005 f

1
f 950
j1900 ( t 0.00025)
2

sinc 30 ( t 0.00025 ) e
30
60

Similarly
1
f + 950
2
j1900 t

sinc ( 30t ) e
30
60
e j 0.0005 f

1
f + 950
j1900 ( t 0.00025 )
2

sinc 30 ( t 0.00025 ) e
30
60

Therefore,
y (t ) = sinc 2 30 ( t 0.00025 ) e j1900 ( t 0.00025) + e j1900 ( t 0.00025)
= 2sinc 2 30 ( t 0.00025 ) cos 1900 ( t 0.00025 )
2.26 The signal 2e 2t u (t ) is input to an ideal LP filter with passband edge frequency
equal to 5 Hz. Find the energy density spectrum of the output of the filter. Calculate
the energy of the input signal and the output signal.
Solution:

41
2012 by McGraw-Hill Education. This is proprietary material solely for authorized instructor use. Not authorized for sale or distribution in any
manner. This document may not be copied, scanned, duplicated, forwarded, distributed, or posted on a website, in whole or part.

Ex =

e 4t
2e u (t ) dt = 4e dt = 4
4
0
2

2 t

x(t ) = 2e 2t u (t )

4 t

=1
0

2
2 + j 2 f
2

The energy density spectrum of the output y(t), Y ( f ) , is related to the energy
density spectrum of the input x(t)
2

2
f
Y( f ) = H( f ) X( f ) =
10 2 + j 2 f
2

Ey =

y (t ) dt =
2

Y( f )

df =

2
f
df

10 2 + j 2 f

1
1
df = 2
df
=
1 + j f
1+ 2 f 2
0
5

Making change of variables f = u df =


Ey =

1+ u

du =

tan 1 ( u )

5
0

du , we obtain

(1.507 ) = 0.9594

Thus output of the LP filter contains 96% of the input signal energy.
2.27 Calculate and sketch the power spectral density of the following signals. Calculate
the normalized average power of the signal in each case.
a. x(t ) = 2 cos (1000 t / 2 ) cos (1850 t + / 4 )
x1 ( t )

x2 ( t )

Solution:
T /2

1
Rx ( ) = lim
x(t ) x(t )dt = Rx1 ( ) + Rx2 ( ) Rx1x2 ( ) Rx2 x1 ( )
T T
T /2
Now

42
2012 by McGraw-Hill Education. This is proprietary material solely for authorized instructor use. Not authorized for sale or distribution in any
manner. This document may not be copied, scanned, duplicated, forwarded, distributed, or posted on a website, in whole or part.

T /2

1
Rx1 ( ) = lim
x1 (t ) x1 (t )dt
T T
T /2
1

2 cos 1000 t 2 cos 1000 ( t ) dt


T T
2
2

T /2
T /2

= lim

T /2

1
cos (1000 ) + cos 1000 ( 2t ) dt
T T
T /2

= 2 lim

= 2 cos (1000 )

The second term is zero because it integrates a sinusoidal function over a


period. Similarly,
T /2

1
1
x2 (t ) x2 (t )dt = cos (1850 )

T T
2
T / 2

Rx2 ( ) = lim

The cross-correlation term


T /2

T /2

1
1
x1 (t ) x2 (t ) = lim
2 cos (1000 t / 2 ) cos 1850 ( t ) + / 4 dt

T T
T T
T / 2
T / 2

Rx1 x2 ( ) = lim

T /2

1
= lim
{cos (850 t 1850 + 3 / 4 ) + sin ( 2850 t 1850 / 4 )}dt
T T
T / 2
is zero because it integrates a sinusoidal function over a period in each case.
Similarly, it can be shown that all other cross-correlation terms are zero.
Therefore,
1
Rx ( ) = Rx1 ( ) + Rx2 ( ) = 2 cos (1000 ) + cos (1850 )
2

Gx ( f ) = {Rx ( )} = 2 cos (1000 ) + cos (1850 )


2

1
= [ ( f 500) + ( f + 500) ] + [ ( f 925) + ( f + 925) ]
4
The normalized average power is obtained by using (2.172) as

43
2012 by McGraw-Hill Education. This is proprietary material solely for authorized instructor use. Not authorized for sale or distribution in any
manner. This document may not be copied, scanned, duplicated, forwarded, distributed, or posted on a website, in whole or part.

[ ( f 500) + ( f + 500)] df + 4 [ ( f 925) + ( f + 925)] df

Gx ( f )df =

Px =

1 1 5
= 1+1+ + =
4 4 2

Power Spectral Density

0.8

0.6

0.4

0.2

0
-1

-0.8

-0.6

-0.4

-0.2
0
0.2
Frequency (kHz)

0.4

0.6

0.8

b. x(t ) = [1 + sin(200 t ) ] cos(2000 t )


Solution:

x(t ) = [1 + sin(200 t ) ] cos(2000 t )


= cos(2000 t ) + sin(200 t ) cos(2000 t )
1
1
= cos(2000 t ) + sin ( 2200 t ) sin (1800 t )
2
2
x (t )
1

x2 ( t )

x3 ( t )

Now
T /2

1
Rx ( ) = lim
x(t ) x(t )dt = Rx1 ( ) + Rx2 ( ) + Rx3 ( )
T T
T /2
where
1
cos ( 2000 )
2
1
Rx2 ( ) = cos ( 2200 )
8
1
Rx3 ( ) = cos (1800 )
8

Rx1 ( ) =

44
2012 by McGraw-Hill Education. This is proprietary material solely for authorized instructor use. Not authorized for sale or distribution in any
manner. This document may not be copied, scanned, duplicated, forwarded, distributed, or posted on a website, in whole or part.

Therefore,
1
1
1
Rx ( ) = cos ( 2000 ) + cos ( 2200 ) + cos (1800 )
2
8
8
1
1
1

Gx ( f ) = {Rx ( )} = cos ( 2000 ) + cos ( 2200 ) + cos (1800 )


8
8
2

1
1
= [ ( f 1000) + ( f + 1000) ] + [ ( f 1100) + ( f + 1100) ]
4
16
1
+ [ ( f 900) + ( f + 900) ]
16

The normalized average power is obtained by using (2.172) as

1
1
Px = Gx ( f )df = [ ( f 1000) + ( f + 1000) ] df +
[ ( f 1100) + ( f + 1100)] df
4
16

1
1 1 1 3
[ ( f 900) + ( f + 900)] df = + + =

16
2 8 8 4

Power Spectral Density

0.25

0.2

0.15

0.1

0.05

c.

-1

-0.8

-0.6

-0.4

-0.2
0
0.2
Frequency (kHz)

0.4

0.6

0.8

x(t ) = cos 2 ( 200 t ) sin (1800 t )

Solution:

45
2012 by McGraw-Hill Education. This is proprietary material solely for authorized instructor use. Not authorized for sale or distribution in any
manner. This document may not be copied, scanned, duplicated, forwarded, distributed, or posted on a website, in whole or part.

1
1
1
1 + cos ( 400 t ) sin (1800 t ) = sin (1800 t ) + sin (1800 t ) cos ( 400 t )
2
2
2
1
1
1
= sin (1800 t ) + sin (1400 t ) + sin ( 2200 t )
2
4
4

x(t ) =

Now

Rx ( ) = Rx1 ( ) + Rx2 ( ) + Rx3 ( )


where
1
Rx1 ( ) = cos (1800 )
8

Rx2 ( ) =

1
cos (1400 )
32

Rx3 ( ) =

1
cos ( 2200 )
32

Therefore,
1
1
1
Rx ( ) = cos (1800 ) + cos (1400 ) + cos ( 2200 )
8
32
32

1
1
1

Gx ( f ) = {Rx ( )} = cos (1800 ) + cos (1400 ) + cos ( 2200 )


32
32
8

1
1
= [ ( f 900) + ( f + 900) ] + [ ( f 700) + ( f + 700) ]
16
64
1
+ [ ( f 1100) + ( f + 1100) ]
64
The normalized average power is obtained by using (2.172) as

1
1
Px = Gx ( f )df =
[ ( f 900) + ( f + 900)]df + [ ( f 700) + ( f + 700)] df

16
64

1
1 1
1
3
[ ( f 1100) + ( f + 1100)] df = + + =

64
8 32 32 16

46
2012 by McGraw-Hill Education. This is proprietary material solely for authorized instructor use. Not authorized for sale or distribution in any
manner. This document may not be copied, scanned, duplicated, forwarded, distributed, or posted on a website, in whole or part.

0.1
0.09

Power Spectral Density

0.08
0.07
0.06
0.05
0.04
0.03
0.02
0.01
0

-1

-0.8

-0.6

-0.4

-0.2

0.2

0.4

0.6

0.8

47
2012 by McGraw-Hill Education. This is proprietary material solely for authorized instructor use. Not authorized for sale or distribution in any
manner. This document may not be copied, scanned, duplicated, forwarded, distributed, or posted on a website, in whole or part.

Das könnte Ihnen auch gefallen